Find P and C such that ##A=PCP^{-1}##

  • Thread starter Thread starter STEMucator
  • Start date Start date
Click For Summary
SUMMARY

The discussion focuses on finding an invertible matrix P and a matrix C of the form [[a, -b], [b, a]] such that A = PCP-1 for the matrix A = [[1, -2], [1, 3]]. The eigenvalues of A are λ1 = 2+i and λ2 = 2-i, with corresponding eigenvectors v1 = [-1+i, 1]t and v2 = [-1-i, 1]t. The matrices derived are C = [[2, -1], [1, 2]] and P = [[-1, -1], [1, 0]], with P-1 = [[0, 1], [-1, -1]]. The theorem discussed confirms that only the eigenvalue λ = a - bi is necessary for constructing C and P.

PREREQUISITES
  • Understanding of complex eigenvalues and eigenvectors
  • Familiarity with matrix inversion techniques
  • Knowledge of the Jordan form and diagonalization of matrices
  • Basic linear algebra concepts, particularly regarding 2x2 matrices
NEXT STEPS
  • Study the implications of complex eigenvalues in linear transformations
  • Learn about the Jordan canonical form for matrices with complex eigenvalues
  • Explore the derivation and application of the theorem regarding A = PCP-1
  • Investigate the properties of eigenvectors and their relationships in complex conjugate pairs
USEFUL FOR

Students of linear

STEMucator
Homework Helper
Messages
2,076
Reaction score
140

Homework Statement



Find an invertible matrix ##P## and a matrix ##C## of the form ##\left( \begin{array}{cc} a & -b \\ b & a \end{array} \right)## such that ##A=PCP^{-1}## when ##A = \left( \begin{array}{cc} 1 & -2 \\ 1 & 3 \end{array} \right)##.

Homework Equations



Eigenvalues for A:

##\lambda_1 = 2+i##
##\lambda_2 = 2-i##

Eigenvectors for eigenvalues:

##v_1 = [-1 + i, 1]^t##
##Re(v_1) = [-1, 1]^t##
##Im(v_1) = [1, 0]^t##

##v_2 = [-1 - i, 1]^t##
##Re(v_2) = [-1, 1]^t##
##Im(v_2) = [-1, 0]^t##

A theorem:

Let ##A## be a real 2x2 matrix with a complex eigenvalue ##\lambda = a - bi, (b≠0)## and an associated eigenvector ##v##.

Then ##A=PCP^{-1}## where ##P = [ Re(v), Im(v) ]## and ##C = \left( \begin{array}{cc} a & -b \\ b & a \end{array} \right)##

The Attempt at a Solution



I've basically solved this, but I had a question.

Does the theorem state that I only care about the eigenvalue ##a - bi##? In this case it would be ##\lambda_2 = 2-i##. If that's the case, ##\lambda_1## is useless in finding the required matrices.

Granted that ##\lambda_1## and ##v_1## have nothing to do with the problem, the matrices required are:

##C = \left( \begin{array}{cc} 2 & -1 \\ 1 & 2 \end{array} \right)##
##P = \left( \begin{array}{cc} -1 & -1 \\ 1 & 0 \end{array} \right)##
##P^{-1} = \left( \begin{array}{cc} 0 & 1 \\ -1 & -1 \end{array} \right)##

Where I have plugged the magnitudes of ##a## and ##b## into ##C##.

My real curiosity lies in why I only care about the eigenvalue ##a - bi##?
 
Physics news on Phys.org
Complex eigenvalues come in pairs, so if you know one of them, the other is uniquely determined. Also notice that the first components of your eigenvectors are conjugates of one another. Notice also that the real parts of your two eigenvectors are the same, and the imaginary parts are scalar multiples of one another.
 
Mark44 said:
Complex eigenvalues come in pairs, so if you know one of them, the other is uniquely determined. Also notice that the first components of your eigenvectors are conjugates of one another. Notice also that the real parts of your two eigenvectors are the same, and the imaginary parts are scalar multiples of one another.

This much I have already seen. I was more curious as to why the eigenvalue ##a+bi## is being neglected in the theorem. Is it solely due to the definition of ##C##?
 
I'll bet the theorem could be rewritten so that it uses a + bi instead of a - bi.
 
  • Like
Likes 1 person
Mark44 said:
I'll bet the theorem could be rewritten so that it uses a + bi instead of a - bi.

Indeed, I conjecture that:

Let ##A## be a real 2x2 matrix with a complex eigenvalue ##\lambda = a + bi, (b≠0)## and an associated eigenvector ##v##.

Then ##A=PCP^{-1}## where ##P = [ Re(v), Im(v) ]## and ##C = \left( \begin{array}{cc} a & b \\ -b & a \end{array} \right)##

I have verified the application of this theorem. I'll go prove it in a more general setting now.

Thank you for your help.
 
Question: A clock's minute hand has length 4 and its hour hand has length 3. What is the distance between the tips at the moment when it is increasing most rapidly?(Putnam Exam Question) Answer: Making assumption that both the hands moves at constant angular velocities, the answer is ## \sqrt{7} .## But don't you think this assumption is somewhat doubtful and wrong?

Similar threads

  • · Replies 3 ·
Replies
3
Views
2K
  • · Replies 4 ·
Replies
4
Views
1K
Replies
6
Views
1K
  • · Replies 7 ·
Replies
7
Views
2K
  • · Replies 2 ·
Replies
2
Views
1K
  • · Replies 9 ·
Replies
9
Views
2K
Replies
8
Views
3K
  • · Replies 6 ·
Replies
6
Views
2K
  • · Replies 5 ·
Replies
5
Views
2K
  • · Replies 1 ·
Replies
1
Views
2K